Hello, dear friend, you can consult us at any time if you have any questions, add WeChat: daixieit

MATH2215 Mathematical Analysis

Assignment 2

Scan your assignment into a single PDF file.  Submit it through Moodle before 7:00p.m. on Feb.  16, 2022 (Wednesday).

You may use any result covered in lecture notes from Chapter 0 to Chapter 1 and the following theorem and definition for this assignment. However, if you use anything else, you must include in a full proof of the result that you are using.


⃞Bythe axiom of completeness, we have a known theorem:

Such y will be denoted as x1/n = . Problem 8 is an example to verify this theorem.

 

= x .


1. Without using binomial theorem show the following problems:

(a) Show that 1 > 0 and − 1 < 0.  [Note that a > 0 that means a ∈ P, the positive subset.  Hint: Use

Theorem 8.6.C (e) and Definition 8.6.B.]

(b) Suppose a > 0.  Show that  > 0.  (You may use the property in the field that   0.)  [Hint: Use

the law of trichotomy.]

(c) Suppose b > a > 0. Show that 0 <  <  (you may consider  − ).

(d) Suppose b > a > 0. Use induction on n to show that bn > an, where n ∈ N.

(e) Suppose x > 1. Show that xn+1  > xn  > 1 for n ∈ N. Hence xm  > xn, for m, n ∈ N and m > n. The

last part is not required to show.

(f) Show that if x > 1 and r, s are positive rational numbers with r < s, then xr  < xs .

2. Write a formal definition in the style of Definition 1.3.2 for the infimum or the greatest lower bound of a set.

3. Mark each statement True or False. Justify your answer if the statement is false.

(a) Every nonempty subset of N contains a minimum.

(b) Every nonempty subset of N contains a maximum.

(c) Between any two unequal rational numbers there is an irrational number.

4. Compute, without proofs, the supremum, infimum, maximum and minimum of the following sets, if they exist. Otherwise, write “none”.

(a)  {1 −  | n ∈ N},


(b)

 

(c)


n=1

n=1


[ , 2 − ] ,

(1 −  , 1 + ).


5. Let A ⊂ R be bounded below, and let c ∈ R. Define the sets c + A and cA by c + A = {c + a | a ∈ A} and cA = {ca | a ∈ A}.

(a) Show that inf(c + A) = c + inf A.

(b) For c ≥ 0, show that inf(cA) = c inf A.

6. Prove that  ∩ (0, ) = ∅. Note that, this demonstrates that the intervals in the Nested Interval Property

 

7. Let I stand for the set of irrational numbers, i.e., I = Qc = R \ Q.

(a) Show that if a, b ∈ Q, then ab and a + b are elements of Q as well.

(b) Show that if a ∈ Q and t ∈ I, then a + t ∈ I and at ∈ I as long as a  0.

(c) Justify the statement.  “If x, y ∈ I, then xy ∈ I.”

8. Prove that there is a positive number α such that α3 = 5.

[Hint: Let T = {x ∈ R | x3 < 5}. Apply a similar argument of the proof of Theorem 1.4.5. on T.]